Skip to Main Content

PrepTest 73, Game 2, Question 13

Transcript

Question 13. This is a rule change question specifically unequivalent rule question. These are definitely questions you wanna leave till the end of the game even if they weren't the last question like Here and like most games they appear in. The reason is that most of the time they take a little bit longer than other question types, and so you're going to be getting fewer points for the amount of time you're investing in them.

That means they're good candidates for skipping if you need to keep your forward momentum going. Generally we say they're not great targets unless you're in the 160s. And then you might need to start to learn how to do them well and quickly. Equivalent rule question asked you to find a rule listed in the answers that could be substituted into the game and play.

Ladies have a rule that you have been using throughout the game. Essentially you're gonna be crossing out the original rule and the new rule could be written into the place of that original rule and when it is, it won't change the game in any way. It would create the exact same game that you have had the whole time, which means that everything the original rule allowed will still be allowed.

Out everything the original rule block will still be blocked. The game will be unchanged by swapping in this new rule. Usually with these questions, we try to see what we can eliminate from previous work first. Because if a rule is going to be equivalent to something that we've been using throughout the game, it'll have to have been true throughout the entire game.

Throughout all the questions that we've done to this point, so we can check each answer choice against our sketches to see if it's always been true and if it hasn't always been true, we can eliminate it. In this game that's only gonna get rid of one answer choice for us, answer choice A Is not true in the sketch for question 10. Answer choice A says L's speech has to be at three, but we've seen L at 2.

The other answer choices unfortunately have all been true to this point. Answer choice says that M's speech can't be earlier than Z, and since Z was one and that was true. Nobody gets to be earlier than Z. So I don't know if that rule is equivalent answer choice C similar says that either X or Y must be after Z.

And since Z is in the 1:00 hour We know that at least one of those will have to be after him, because there's only one spot at 1 that anybody else could take. Answer choice (D) says either X or Y's speech has to be at 2 and that's been true in pretty much every sketch we've done so far and finally answer choice (E) says that Z's speech has to be at 1, which Which was our deduction, so, yes, it has always been true.

So what we have right now are a bunch of question marks. Now since answer choice E was one of our deductions, though, it does suggest that when we go do the second half of the question, We might want to start with it. And the second half of the question would be to try to test answer choices. And the way that we test answer choices as we think about trying to break the rule that we're being asked to replace, while following the rule that we're given in the answer choice.

If the answer choice lets you break the rule that you started with. It's not equivalent to the rule that you started with. And when we think about that, will realize that answer choice he has to be the answer because it z has to be at 1, there's no way we're going to be able to put either X or Y in front of Z. This new rule will force us to follow the original rule essentially it will recreate the original rule, which means the answer choice He is our answer.

But what if we hadn't noticed that? What if we'd started testing at the top with answer choice B, our tests would go something like this answer choice B says that m can be before z. So if we place them both at 2PM, and won't be in front of z, he'll be at the same time. And that would according to rule one force l to go to three and then that would mean that x&y could both slot into 1pm.

essentially creating a sketch that looks like this. And in that sketch, we are following the rule from answer choice B, because M is not in front of z, it's at the same time as z, but we're violating the original rule because both. X and Y are in front of Z. So that means that answer choice B is not our answer.

If we were to test answer choice C, it would work similarly, indeed, you could just switch Y from 1 PM back to 3 PM, which moves M and L up and then we'd be able to follow this rule. Because the rule says that either x or y has to be later than z. So if we do that here, we would have y later than z. So we'd be following the new rule from answer choice C, but we would be violating our original rule because x is in front of z.

And then finally, if we look at answer choice D, which says that either X or Y Z speech has to be at to overtake the sketch that we just did and move why up to two, then we can follow this new rule. Because either X or Y is it 2 in this case, Y is it 2 but we are still violating the original rule that you can't have x or y in front of z. So since we can violate the original rule while following the new one, answer choice D is new rule is not a replacement for the original rule, and that would leave us with E.

So now you can see why it is that we recommend skipping these without a good bit of skill and a little bit of luck, it might take quite some time to do them. So put them off till the end and if you don't have time, skip them. But that is the last question in the game. We are done with game two.

Read full transcript